Akademisyenler öncülüğünde matematik/fizik/bilgisayar bilimleri soru cevap platformu
2 beğenilme 0 beğenilmeme
1.6k kez görüntülendi
$\sum\limits_{n=1}^{\infty}\frac{\sin n}{n}$ serisinin karakterini belirleyiniz. (yakinsak-iraksak)

Bu soruyu değerini bulunuz olarak değiştiriyorum.
Lisans Matematik kategorisinde (25.3k puan) tarafından 
tarafından düzenlendi | 1.6k kez görüntülendi
Bu soru sanıyorum, Cauchy ile bulunabilir. Ancak şu an sadece bir düşünce, biraz düşüneceğim.

5 Cevaplar

1 beğenilme 0 beğenilmeme

KONRAD KNOPP, THEORY AND APPLICATION OF INFINITE SERIES,

BLACKIE & SON LIMITED, 1954.

Sorunun cevabı yukarıdaki kitabın  375. sayfasında bulunabilir. Kitabın Fourier serileri kısmında çok daha fazla bilgi var.

\[
\sum_{n=1}^{\infty }\frac{\sin nx}{n}=\left\{
\begin{array}{ccc}
\frac{\pi -x}{2} &  & 0<x<2\pi  \\
0 &  & x=0,2\pi
\end{array}
\right.
\]

(541 puan) tarafından 
1 beğenilme 0 beğenilmeme
Ben şöyle bi çözüm yapıyorum. yanlışlarım eksiklerim varsa lütfen uyarın beni..

$\dfrac{\sin(n)}{n}=\dfrac{e^{in}-e^{-in}}{2in}$ olmak üzere

 

$\displaystyle\sum_{n=1}^{\infty}\frac{\sin(n)}{n}=\sum\limits_{n=1}^{\infty}\dfrac{e^{in}-e^{-in}}{2in}={-\frac{i}{2}}{\sum\limits_{n=1}^{\infty}\frac{e^{in}-e^{-in}}{n}}={-\frac{i}{2}}\left[\sum\limits_{n=1}^{\infty}\frac{e^{in}}{n}-\sum\limits_{n=1}^{\infty}\frac{e^{-in}}{n}\right]$

 

$-\ln(1-x)=\displaystyle\sum_{n=1}^{\infty}{\frac{x^n}{n}}\implies$

$\boxed{-\ln(1-e^{i})=\displaystyle\sum_{n=1}^{\infty}{\frac{(e^{i})^n}{n}}}$  ve $\boxed{-\ln(1-e^{-i})=\displaystyle\sum_{n=1}^{\infty}{\frac{(e^{-i})^n}{n}}}$  olmak üzere

 

$\displaystyle\sum\limits_{n=1}^{\infty}\frac{\sin(n)}{n}={-\frac{i}{2}}\left[{\sum\limits_{n=1}^{\infty}\frac{e^{in}}{n}}-{\sum\limits_{n=1}^{\infty}\frac{e^{-in}}{n}}\right]={\frac{i}{2}}\left[\ln(1-e^{i})-\ln(1-e^{-i})\right]=\frac{i}{2}\ln\left[\dfrac{1-e^{i}}{1-e^{-i}}\right]=\frac{i}{2}\ln\left[-e^{i}\right]$
(470 puan) tarafından 
tarafından düzenlendi
1 beğenilme 0 beğenilmeme

Sadece yakınsaklığının gösterilişi.

Bir teorem kullanarak kolayca yakınsak olduğu gösterilebilir.

Teorem: $\sum a_n$ kısmi toplamları sınırlı bir seri ve $(b_n)$ azalan ve limiti 0 olan bir dizi olsun.

O zaman, $\sum a_nb_n$ serisi yakınsaktır.

(https://kurser.math.su.se/pluginfile.php/38714/mod_resource/content/1/serietentan.pdf sayfa 3 de daha genel şekli var)

Tek yapmamız gereken $\sum \sin n$ serisinin kısmi toplamlarının sınırlı olduğunu göstermektir.

Bu da $\sin n=Im(e^{in})$ oluşundan ve geometrik seri toplam formülünden elde edilir:

$|\sin1+\sin2+\cdots+\sin n|=|Im(e^{i}(1+e^i+\cdots+e^{(n-1)i})|=\left|Im\left(e^i\frac{e^{ni}-1}{e^i-1}\right)\right|\leq\left|\frac{e^{ni}-1}{e^i-1}\right|<\frac2{|e^i-1|}$ den, 

$\sum \sin n$ serisinin kısmi toplamların sınırlı olduğu görülür.

 

(6.1k puan) tarafından 
ece çelik yukarda benzerini yapmış toplamını da bulmuş.
Aslinda istedigim analiz 1 kivaminda bir ispatti. iIlk kisim bunu yansitiyor.
Bu soru (herhalde yanlışlıkla konmuş) Tuğrul Taner in (ODTÜ de) Calculus kitabında vardı O zaman çözemediğimi hatırlıyorum.

Ikinci kismi icin de: (sin n toplaminin sınırlı olması)
https://matkafasi.com/129592

Buradaki yöntem bağlantıdaki soruya cevap olarak da eklenebilir.
$\sum_{n=1}^\infty \frac{\sin n}n$ serisi mutlak yakınsak mıdır?
0 beğenilme 0 beğenilmeme

hocam karsilastirma testi vardi an<cn olmak uzere 

cn yakinsaksa an de yakinsaiktir 

sinn/n<1/n den bu nedenle cn=1/n aliriz 1/n diziside harmonik seri oldugundan iraksaktir istisna idi bu

(84 puan) tarafından 

Çözümde eksiklikler ve yanlışlıklar var. İlk olarak pozitif tanımlı olmadığından kıyaslayamayız lakin mutlak değerini kıyaslayabiliriz. Eğer bir seri üsten sınırlı değilse zaten ıraksaktır. Eğer üstten sınırlı ise  (pozitif tanımlı (aslında olmasına da gerek yok)) üsten sınırının sonsuza kadar toplamı da  ıraksaktır (eger sıfır değilse ki pozitif tanımlılarda sıfır olması sadece hepsinin sıfır oldugu durumda olur), yani istisnai değil, sadece yanlış seçim. 

Eğer konuya yeni başlayan biri iseniz ters örnekleri bulmanızı tavsiye ederim, bu anlamanıza daha çok yardımcı olur kanaatindeyim.

0 beğenilme 0 beğenilmeme

Dikkat: Cozum hakkinda suphelerim var.

 

$f(x)=\dfrac{\sin(x)}{x}$ azalan  bir fonksiyon olmakla birlikte bir  $k\in \mathbb{N^+ }$ icin $f(x)\geq0 \in [k,\infty)$ olmuyor. Bu da integral testinin taniminda gecen kosullari ihlal ediyor.

 

Bununla birlikte sonlu toplamlar icin esitsizlik saglaniyor. Yani

 

$\displaystyle\lim_{N\to\infty }\left(\sum_{n=1}^{N}\dfrac{\sin(n)}{n}\right)=\displaystyle\lim_{N\to\infty }\left(\displaystyle\sum_{n=0}^N\dfrac{\sin(n)}{n}-1\right)<\displaystyle\lim_{N\to\infty }\left(\displaystyle\int_0^{N}\dfrac{\sin(x)}{x}dx\right),$  $\forall \;N\in\mathbb{N^+}$    dir.


-----------------------------------------------------------------------------------------------------------------------

Cozum:

 

Oncelikle serimizi $n=0$'dan baslatabilirmiyiz onu gosterelim.

 

$\displaystyle\lim_{x\to0}\dfrac{\sin(x)}{x}=\dfrac00\underset{L'H}\implies\displaystyle\lim_{x\to0}\dfrac{\cos(x)}{1}=1\implies\displaystyle\lim_{n\to0}\dfrac{\sin(n)}{n}=1$

 

$\displaystyle\sum_{n=1}^{\infty}\dfrac{\sin(n)}{n}=\displaystyle\sum_{n=0}^{\infty}\dfrac{\sin(n)}{n}-1$

 

$\displaystyle\int_0^{\infty}\dfrac{\sin(x)}{x}dx=\dfrac{\pi}{2} \quad \text{yakinsak oldugundan }\implies\displaystyle\sum_{n=0}^{\infty}\dfrac{\sin(n)}{n}$ integral testine gore yakinsaktir $\implies\displaystyle\sum_{n=1}^{\infty}\dfrac{\sin(n)}{n}$ yakinsaktir.

 

Not:

 

$\displaystyle\sum_{n=1}^{\infty}\dfrac{\sin(n)}{n}=\displaystyle\sum_{n=0}^{\infty}\dfrac{\sin(n)}{n}-1<\displaystyle\int_0^{\infty}\dfrac{\sin(x)}{x}dx=\dfrac{\pi}{2}\implies\displaystyle\sum_{n=1}^{\infty}\dfrac{\sin(n)}{n}<1+\dfrac{\pi}{2}$ usten sinirli oldugunu soyleyebiliriz.

 

-----------------------------------------------------------------------------------------------------------------------

$\displaystyle\int_0^{\infty}\dfrac{\sin(x)}{x}dx=\dfrac{\pi}{2}$        Internette bir cok gosterimi var. Ben asagidakini sevdim ve umarsizca  asirdim. Laplace donusumlerinden faydalanilmistir.

 

$$I(s)=\displaystyle\int_0^\infty e^{-sx}\frac{\sin x}{x}dx=\displaystyle\lim_{b\to\infty}\int_0^be^{-sx}\frac{\sin x}{x}dx, s\ge 0.$$ $s$'ye gore turev alirsak

 

$$I'(s)=-\displaystyle\lim_{b\to\infty}\int_0^b e^{-sx}\sin xdx\underset{\text{*}}=\displaystyle\lim_{b\to\infty}\frac{e^{-sx}}{s^2+1}(s\sin x+\cos x)\Big|_0^b=-\frac{1}{s^2+1}.$$

*iki defa parcali integrasyon

$$I'(s)=-\frac{1}{s^2+1}.$$ integral alirsak

 

$$I(\infty)-I(0)=-\int_0^\infty\frac{1}{s^2+1}ds=-\frac{\pi}{2}.$$

$I(\infty)=0$ gosterimi okuyucuya birakilmistir.

$$-I(0)=-\frac{\pi}{2}\implies I(0)=\displaystyle\int_0^{\infty}\dfrac{\sin(x)}{x}dx=\dfrac{\pi}{2}$$

 

Kaynakca: https://math.stackexchange.com/questions/5248/evaluating-the-integral-int-0-infty-frac-sin-x-x-mathrm-dx-frac-pi

(2.9k puan) tarafından 
tarafından düzenlendi
İntegral Testini bu toplam için uygulayabilir miyiz?
Ben de soruyu duzenliyordum.. Dun paylastiktan sonra integral testini uygulayamabilecegimi farkettim ve gizledim soruyu, bugun ise duzeltme ile yeniden actim ve hala bir umut tasiyorum :)
20,200 soru
21,726 cevap
73,275 yorum
1,887,809 kullanıcı